Difference between revisions of "2011 AMC 12B Problems/Problem 25"

(Solution)
m (Solution)
Line 65: Line 65:
  
 
<br />
 
<br />
If <math> \left[\frac{100}{k}\right] </math> got round down, then <math>1 \le n \le \frac{1}{2}</math> all satisfy the condition along with <math>n = k</math>, which makes  
+
If <math> \left[\frac{100}{k}\right] </math> got round down, then <math>1 \le n \le \frac{k}{2}</math> all satisfy the condition along with <math>n = k</math>, which makes  
  
 
<math>P(k) \ge \frac{1}{2} + \frac{1}{2k}</math>.
 
<math>P(k) \ge \frac{1}{2} + \frac{1}{2k}</math>.
  
 
<br />
 
<br />
If <math> \left[\frac{100}{k}\right] </math> got round up, then <math>\frac{1}{2} \le n \le \frac{1}{2}</math> all satisfy the condition along with <math>n = 1</math>, which makes  
+
If <math> \left[\frac{100}{k}\right] </math> got round up, then <math>\frac{k}{2} \le n \le k</math> all satisfy the condition along with <math>n = 1</math>, which makes  
  
 
<math>P(k) \ge \frac{1}{2} + \frac{1}{2k}</math>.
 
<math>P(k) \ge \frac{1}{2} + \frac{1}{2k}</math>.

Revision as of 10:18, 11 March 2011

Problem

For every $m$ and $k$ integers with $k$ odd, denote by $\left[\frac{m}{k}\right]$ the integer closest to $\frac{m}{k}$. For every odd integer $k$, let $P(k)$ be the probability that

\[\left[\frac{n}{k}\right] + \left[\frac{100 - n}{k}\right] = \left[\frac{100}{k}\right]\]

for an integer $n$ randomly chosen from the interval $1 \leq n \leq 99!$. What is the minimum possible value of $P(k)$ over the odd integers $k$ in the interval $1 \leq k \leq 99$?

$\textbf{(A)}\ \frac{1}{2} \qquad \textbf{(B)}\ \frac{50}{99} \qquad \textbf{(C)}\ \frac{44}{87} \qquad \textbf{(D)}\  \frac{34}{67} \qquad \textbf{(E)}\  \frac{7}{13}$

Solution

Answer: $(D)  \frac{34}{67}$


First of all, you have to realize that

if $\left[\frac{n}{k}\right] + \left[\frac{100 - n}{k}\right] = \left[\frac{100}{k}\right]$

then $\left[\frac{n - k}{k}\right] + \left[\frac{100 - (n - k)}{k}\right] = \left[\frac{100}{k}\right]$

So, we can consider what happen in $1\le n \le k$ and it will repeat. Also since range of $n$ is $1$ to $99!$, it is always a multiple of $k$. So we can just consider $P(k)$ for $1\le n \le k$.


LET $\text{fpart}(x)$ be the fractional part function

This is an AMC exam, so use the given choices wisely. With the given choices, and the previous explanation, we only need to consider $k = 99$, $87$, $67$, $13$. $1\le n \le k$


For $k > \frac{200}{3}$, $\left[\frac{100}{k}\right] = 1$. 3 of the $k$ that should consider lands in here.

For $n < \frac{k}{2}$, $\left[\frac{n}{k}\right] = 0$, then we need $\left[\frac{100 - n}{k}\right] = 1$

else for $\frac{k}{2}< n < k$, $\left[\frac{n}{k}\right] = 1$, then we need $\left[\frac{100 - n}{k}\right] = 0$

For $n < \frac{k}{2}$, $\left[\frac{100 - n}{k}\right] = \left[\frac{100}{k} - \frac{n}{k}\right]= 1$

So, for the condition to be true, $100 - n > \frac{k}{2}$ . ( $k > \frac{200}{3}$, no worry for the rounding to be $> 1$)

$100 > k > \frac{k}{2} + n$, so this is always true.

For $\frac{k}{2}< n < k$, $\left[\frac{100 - n}{k}\right] = 0$, so we want $100 - n < \frac{k}{2}$, or $100 < \frac{k}{2} + n$

$100 <\frac{k}{2} + n <  \frac{3k}{2}$

For k = 67, $67 > n > 100 - \frac{67}{2} = 66.5$

For k = 69, $69 > n > 100 - \frac{69}{2} = 67.5$

etc.


We can clearly see that for this case, $k = 67$ has the minimum $P(k)$, which is $\frac{34}{67}$. Also, $\frac{7}{13} > \frac{34}{67}$ .

So for AMC purpose, answer is (D).



Now, let's say we are not given any answer, we need to consider $k < \frac{200}{3}$.

I claim that $P(k) \ge \frac{1}{2} + \frac{1}{2k}$


If $\left[\frac{100}{k}\right]$ got round down, then $1 \le n \le \frac{k}{2}$ all satisfy the condition along with $n = k$, which makes

$P(k) \ge \frac{1}{2} + \frac{1}{2k}$.


If $\left[\frac{100}{k}\right]$ got round up, then $\frac{k}{2} \le n \le k$ all satisfy the condition along with $n = 1$, which makes

$P(k) \ge \frac{1}{2} + \frac{1}{2k}$.

See also

2011 AMC 12B (ProblemsAnswer KeyResources)
Preceded by
Problem 24
Followed by
Last Problem
1 2 3 4 5 6 7 8 9 10 11 12 13 14 15 16 17 18 19 20 21 22 23 24 25
All AMC 12 Problems and Solutions